How Do You Solve Complex Inverse Laplace Transforms?

In summary: After all, inverse Laplace Transforms are linear, so you should know that \mathcal{L}^{-1}\left[\frac{1}{s}+\frac{5}{3(s^2+\frac{5}{3}s+\frac{5}{3})}\right]=\mathcal{L}^{-1}\left[\frac{1}{s}\right]+\mathcal{L}^{-1}\left[\frac{5}{3(s^2+\frac{5}{3}s+\frac{5}{3})}\right]After all, inverse Laplace Transforms are
  • #1
CremeBrulee
8
0
My books aren't clear at running through these inverse laplace transforms and this ones got me snookered. I'm trying to perform the laplace transform of:

1/s + 40/(24s^2 + 40s + 40)

Factor out 24 becomes

1/s + 1.667/(s^2 +1.667s +1.667)

Finding A, B, C

A/s + Bs+C/(s^2 +1.667s +1.667)

A=1
B=-1
C=-1.667

Substituting back into Eq becomes

1/s - (s+1.667)/(s^2 +1.667s +1.667)

However, I'm stuck here as the Eq doesn't look like its in the correct form to use in the inverse laplace tables? Do I need to manipulate denominator to present as (s+...)(s+...) using roots? Roots are found to be -0.833 +/- j0.986

Or split numerator into two parts ie 1/s - (s+1)/(s^2 +1.667s +1.667) + 1.667/(s^2 +1.667s +1.667)

Or both??

1/s - (s+1)/(s+(-0.833)) + j0.986) + 1.667/(s+(-0.833)) - j0.986)

Using the transform table at:

http://www.swarthmore.edu/NatSci/ec...nMethods/LaplaceZTable/LaplaceZFuncTable.html

Item 16 in the table looks like similar format but I believe Item 18 is closer after finding wn=1.29 & zeta=0.646 however, using item 18 doesn't account for the additional s in the numerator??

I feel I'm close but not quite there to finding this solution and its frustrating me! Any help or direction would be much appriciated.
 
Physics news on Phys.org
  • #2
CremeBrulee said:
1/s + 1.667/(s^2 +1.667s +1.667)

Finding A, B, C

A/s + Bs+C/(s^2 +1.667s +1.667)

A=1
B=-1
C=-1.667

I'm not quite sure what you are trying to do here, but it seems like you are attempting to use partial fraction decomposition, without actually decomposing the denominator. Where did you get those values of 'A', B' and 'C' from? It should be clear that A=1, B=0 and C=5/3 reproduces your original expression, so the values you've found are clearly incorrect!

Instead of partial fraction decomposition, try completing the square on the denominator of your [itex]\frac{5}{3(s^2+\frac{5}{3}s+\frac{5}{3})}[/itex] term and then applying the frequency shift rule.

After all, inverse Laplace Transforms are linear, so you should know that

[tex]\mathcal{L}^{-1}\left[\frac{1}{s}+\frac{5}{3(s^2+\frac{5}{3}s+\frac{5}{3})}\right]=\mathcal{L}^{-1}\left[\frac{1}{s}\right]+\mathcal{L}^{-1}\left[\frac{5}{3(s^2+\frac{5}{3}s+\frac{5}{3})}\right][/tex]

and so the only difficulty you should have is with the second term.
 
Last edited:
  • #3
I found A,B,C using partial fraction expansion of second order polynomial where 5/3 = 1.667 (from my earlier post) i.e.

1/s + (5/3)/(s^2 + 5/3s + 5/3) = A/s + (Bs + C)/(s^2 + 5/3s + 5/3)

5/3 = A (s^2 + 5/3s + 5/3) + (Bs + C) s

5/3 = As^2 + 5/3As + 5/3A + Bs^2 + Cs

5/3 = s^2 (A+B) + s (5/3A + C) + 5/3A

0 = s^2 (A + B)
0 = s^1 (5/3A + C)
5/3 = s^0 (5/3A)

Therefore:

5/3 = 5/3A
A = 1

0 = A + B
0 = 1 +B
B = -1

0 = 5/3A + C
0 = 5/3 + C
C = -5/3

A/s + (Bs + C)/(s^2 + 5/3s + 5/3) = 1/s - (s+5/3) /(s^2 + 5/3s +5/3)

Is this method not correct to find A,B,C? I know there are different methods out there but this one seems the quickest to perform.
 
  • #4
CremeBrulee said:
A/s + (Bs + C)/(s^2 + 5/3s + 5/3) = 1/s - (s+5/3) /(s^2 + 5/3s +5/3)

Is this method not correct to find A,B,C? I know there are different methods out there but this one seems the quickest to perform.

No, it's not correct.

For starters, the point of partial fraction decomposition is to break compound fractions into simpler fractions... that means that you want to break factions with denominators of order 2 or higher into fractions with denominators of lower order. What you're trying to do here:

1/s + (5/3)/(s^2 + 5/3s + 5/3) = A/s + (Bs + C)/(s^2 + 5/3s + 5/3)

Takes a fraction with a quadratic denominator and decomposes it into another fraction with a quadratic denominator...what's the use of that?

Not only that, but the values of A,B and C you come up with are incorrect...just plug in s=1 to your original expression, and your final expression...do you get the same number?

Instead of using partial fractions, just complete the square on the denominator of your second term
 

Related to How Do You Solve Complex Inverse Laplace Transforms?

1. What is the Inverse Laplace Transform?

The Inverse Laplace Transform is a mathematical operation that allows us to convert a function from the Laplace domain to the time domain. It is the inverse operation of the Laplace Transform and is used to solve differential equations and analyze signals and systems.

2. How is the Inverse Laplace Transform calculated?

The Inverse Laplace Transform is calculated by using a table of known Laplace transforms, along with techniques such as partial fraction decomposition and the use of complex numbers. It involves taking the inverse of the Laplace transform variable s and solving for the original function in the time domain.

3. What is the importance of the Inverse Laplace Transform in science and engineering?

The Inverse Laplace Transform has many applications in science and engineering, particularly in the fields of control systems, signal processing, and electrical circuits. It allows us to analyze and understand the behavior of systems over time, and is essential in solving many differential equations that arise in these fields.

4. Can the Inverse Laplace Transform be used for all Laplace transforms?

In theory, yes, the Inverse Laplace Transform can be used for all Laplace transforms. However, in practice, some Laplace transforms may be too complex to be solved using traditional techniques, and numerical methods may be required to approximate the inverse transform.

5. Are there any limitations or drawbacks to using the Inverse Laplace Transform?

The Inverse Laplace Transform can only be applied to functions that have a Laplace transform. This means that certain types of functions, such as those with discontinuities or infinite discontinuities, may not have a Laplace transform and therefore cannot be solved using this method. Additionally, the calculations involved in finding the inverse transform can be quite complex and time-consuming, and may require the use of advanced mathematical techniques.

Similar threads

  • Calculus and Beyond Homework Help
Replies
10
Views
1K
  • Calculus and Beyond Homework Help
Replies
8
Views
2K
  • Calculus and Beyond Homework Help
Replies
8
Views
2K
Replies
5
Views
1K
  • Calculus and Beyond Homework Help
Replies
2
Views
2K
  • Calculus and Beyond Homework Help
Replies
1
Views
1K
  • Calculus and Beyond Homework Help
Replies
1
Views
1K
  • Calculus and Beyond Homework Help
Replies
4
Views
2K
  • Calculus and Beyond Homework Help
Replies
5
Views
2K
  • Calculus and Beyond Homework Help
Replies
3
Views
1K
Back
Top